Computing representation number quad forms

Click For Summary
The discussion focuses on computing the representation number of two quadratic forms, Q and R, defined by specific matrices. The user successfully determines that r_Q(1) equals 4 by setting u and v to zero, allowing x and y to take values of ±1. However, confusion arises when attempting to solve for r_R(1), where the conditions u + v = 1 and |v| = 0, ±1 are introduced, leading to multiple potential solutions. Participants emphasize the importance of ensuring each term in the quadratic forms remains less than or equal to 1, which clarifies the reasoning behind the conditions for R. The conversation highlights the complexity of solving quadratic forms and the necessity of careful analysis in deriving conditions for integer solutions.
binbagsss
Messages
1,291
Reaction score
12

Homework Statement



## r_{A} (n) = ## number of solutions of ## { \vec{x} \in Z^{m} ; A[\vec{x}] =n} ##
where ##A[x]= x^t A x ##, is the associated quadratic from to the matrix ##A##, where here ##A## is positive definite, of rank ##m## and even. (and I think symmetric?)

I am solving for the ##r_{A}(1) ## for the two quadratic forms:

##Q(x,y,uv)= 2(x^2+y^2+u^2+v^2)+2xu+xv+yu-2yv##
##R(x,y,uv)=x^2+4(y^2+u^2+v^2)+xu+4yu+3yv+7uv##

Homework Equations



see above

The Attempt at a Solution


[/B]
diagonalized these read:

##Q=2(x+u/2+v/4)^2+2(y-v/2+u/4)^2+11u^2/8 + 11v^2/8 ##
##R=(x+1/2u)^2+4(y+1/2u+3/8v)^2+11/4(u+v)^2+11/16v^2 ##

Solving ##Q=1## with all ##x,y,u,v## integer, it is clear that ##u,v=0## is needed, and then ##x,y=\pm 1 ## gives ##r_{Q}(1)=4##.

Now looking at ## r_{R}(1) ## by the same reasoning as above I would have said that we require ##v=0## , and then I' m not sure what to do.

However the solution is:

Must have ##u+v=1 ## & ##|v|=0,\pm 1 ##, this gives ## \pm(1,0,0,0)##, ##\pm(1,0,-1,1) ## , ##\pm(0,0,-1,1)##

(the symbol that I interpreted as '&' in the solutions is a bit smudged, so looking at the solutions I'm not sure that this is supposed to be a 'or'? )

Either way, I'm really confused, unsure where these conditions come from, how to think about this in a logical way...

Many thanks for your help in advance.
 
Physics news on Phys.org
binbagsss said:

Homework Statement



## r_{A} (n) = ## number of solutions of ## { \vec{x} \in Z^{m} ; A[\vec{x}] =n} ##
where ##A[x]= x^t A x ##, is the associated quadratic from to the matrix ##A##, where here ##A## is positive definite, of rank ##m## and even. (and I think symmetric?)

I am solving for the ##r_{A}(1) ## for the two quadratic forms:

##Q(x,y,uv)= 2(x^2+y^2+u^2+v^2)+2xu+xv+yu-2yv##
##R(x,y,uv)=x^2+4(y^2+u^2+v^2)+xu+4yu+3yv+7uv##

Homework Equations



see above

The Attempt at a Solution


[/B]
diagonalized these read:

##Q=2(x+u/2+v/4)^2+2(y-v/2+u/4)^2+11u^2/8 + 11v^2/8 ##
##R=(x+1/2u)^2+4(y+1/2u+3/8v)^2+11/4(u+v)^2+11/16v^2 ##

Solving ##Q=1## with all ##x,y,u,v## integer, it is clear that ##u,v=0## is needed, and then ##x,y=\pm 1 ## gives ##r_{Q}(1)=4##.

Now looking at ## r_{R}(1) ## by the same reasoning as above I would have said that we require ##v=0## , and then I' m not sure what to do.

However the solution is:

Must have ##u+v=1 ## & ##|v|=0,\pm 1 ##, this gives ## \pm(1,0,0,0)##, ##\pm(1,0,-1,1) ## , ##\pm(0,0,-1,1)##

(the symbol that I interpreted as '&' in the solutions is a bit smudged, so looking at the solutions I'm not sure that this is supposed to be a 'or'? )

Either way, I'm really confused, unsure where these conditions come from, how to think about this in a logical way...

Many thanks for your help in advance.
Are you sure there are no typos? I calculated ##R(0,0,-1,1) = \frac{61}{64}##. It could also help to multiply the equations by ##8##, resp. ##16##, which would make the comparisons easier.
 
binbagsss said:
Solving Q=1 with all x,y,u,v integer, it is clear that u,v=0 is needed
True, but not quite trivial.
binbagsss said:
and then x,y=±1
How do you get that? Don't those give Q=4?
binbagsss said:
by the same reasoning as above I would have said that we require v=0
Then you would be wrong. I said it wasn't quite trivial.
As fresh_42 writes, it will help to multiply through the equations to eliminate the fractions.
 
haruspex said:
True, but not quite trivial.

How do you get that? Don't those give Q=4?

Then you would be wrong. I said it wasn't quite trivial.
.

oh right, the reason is that each term needs to be ##\leq 1 ## ?
 
binbagsss said:
oh right, the reason is that each term needs to be ##\leq 1 ## ?
Each term must be no more than 1, but I cannot say whether that is the "reason" you were wrong to conclude v=0 since I do not know how you concluded it. All I can say is that there is a solution with v not 0.
 
Question: A clock's minute hand has length 4 and its hour hand has length 3. What is the distance between the tips at the moment when it is increasing most rapidly?(Putnam Exam Question) Answer: Making assumption that both the hands moves at constant angular velocities, the answer is ## \sqrt{7} .## But don't you think this assumption is somewhat doubtful and wrong?

Similar threads

Replies
1
Views
1K
  • · Replies 14 ·
Replies
14
Views
1K
  • · Replies 4 ·
Replies
4
Views
2K
Replies
5
Views
2K
  • · Replies 8 ·
Replies
8
Views
2K
  • · Replies 12 ·
Replies
12
Views
3K
  • · Replies 105 ·
4
Replies
105
Views
6K
  • · Replies 12 ·
Replies
12
Views
3K
  • · Replies 1 ·
Replies
1
Views
1K
  • · Replies 21 ·
Replies
21
Views
2K